Text ch1-2
Text ch1-2
Text ch1-2
Real Numbers
TOPICS
1. The fundamental theorem of arithmetic.
2. Revisiting irrational numbers.
Note: Prime factorisation of different composite numbers are always different and for each
composite number its prime factors are unique.
Fundamental Theorem of Arithmetic
Statement
Every composite number can be expressed (factorised) as a product of primes,
and this factorisation is unique except for the order in which the prime factors are
written.
For example, prime factors of 30 can be written as:
30 = 2 × 3 × 5 = 2 × 5 × 3 = 3 × 5 × 2 = 3 × 2 × 5 = 5 × 2 × 3 = 5 × 3 × 2
So it is clear that prime factors of 30 can be written in 6 different forms but the
prime numbers occur in the prime factorisation of 30 are same in each case only their
order is different but prime factorisation is unique which is only the conclusion given
by fundamental theorem of Arithmetic.
1
2 S. Chand’s New Mathematics for Class X
Method to Find HCF and LCM of two or more numbers with the help of Fundamental
Theorem of Arithmetic
Step 1: Find prime factorisation of all the numbers given of which HCF/LCM
is required.
Step 2: For finding HCF take the product of the smallest power of each common
prime factor present in the prime factorisation of given numbers whose
HCF is required.
Step 3: For finding LCM take the product of greatest powers of each prime factor
available in the prime factorisation of given numbers.
Applications of Fundamental Theorem of Arithmetic
1. We can find HCF of two or more numbers.
2. We can find LCM of two or more numbers.
3. If a and b are two distinct numbers and their HCF is ‘H’ and LCM is ‘L’ then
a×b=H×L
H¥L H¥L
⇒ a= or b=
b a
a¥b a¥b
or H= or L=
L H
Concept for finding the unit’s place digit in the given expansion
1. If unit’s place digit of any number say ‘x’ is zero then the units place digit in the
expansion of (x)n when n ∈ N will always be zero.
Or
Real Numbers 3
We can say that expansion of any number (x)n can end with zero only if prime
factors of base i.e., ‘x’ has at least one 2 and one 5 both if one 2 and one 5 is not
available in the prime factors of base i.e., x then expansion of (x)n cannot end with
zero.
2. If unit’s place digit of a number say ‘x’ is 1 then unit’s place digit in the expansion
of (x)n when n ∈ N will always be 1.
3. If unit’s place digit of a number say x is 2 then unit’s place digit in the expansion
of (x)n, n ∈ N will be 2 if n = 1, 4 if n = 2, 8 if n = 3, 6 if n = 4 and it is the rule that
after n = 4 unit’s place digits start repeating as under
For n = 5 unit’s place digit will be same as for n = 1
For n = 6 unit’s place digit will be same as for n = 2
For n = 7 unit’s place digit will be same as for n = 3
For n = 8 unit’s place digit will be same as for n = 4 and this process continues.
4. If unit’s place digit of a number say x is 3 then unit’s place digit in the expansion
of (x)n for n ∈ N will be 3 if n = 1, 9 if n = 2, 7 if n = 3, 1 if n = 4 and after n = 4
repetition of unit’s place digit will start as per rule explained in point number 3
for each value of n = 5 and above.
5. If unit’s place digit of a number say x is 4 then unit’s place digit in the expansion
of (x)n, n ∈ N will be 4 if n = 1, 6 if n = 2, 4 if n = 3, 6 if n = 4 i.e., for each odd ‘n’,
unit’s place digit will be 4 and for each even ‘n’, unit’s place digit will be even.
6. If unit’s place digit of a number say x is 5 then unit’s place digit in the expansion
of (x)n, n ∈ N will always be 5.
7. If unit’s place digit in a number say x is 6, then unit’s place digit in the expansion
of (x)n, n ∈ N will always be 6.
8. If unit’s place digit of a number say x is 7 then unit’s place digit in the expansion
of (x)n, n ∈ N will be 7 if n = 1, 9 if n = 2, 3 if n = 3, 1 if n = 4 and usual repetition
will be applicable for each value of n = 5 and above.
9. If unit’s place digit of a number say x is 8 then unit’s place digit in the expansion
of (x)n, n ∈ N will be 8 if n = 1, 4 if n = 2, 2 if n = 3, 6 if n = 4 and usual repetition
for any value of n = 5 and above.
10. If unit’s place digit of a number say x is 9 then unit’s place digit in the expansion
of (x)n, n ∈ N will be, 9 if n = 1, 1 if n = 2, 9 if n = 3, 1 if n = 4 and so on so we can
conclude in this case that unit’s place digit will be 9 for any odd value of n and 1
for any even value of n.
Solved Examples
96 ¥ 404 96 ¥ 404
Also, LCM (96, 404) = = = 9696
HCF (96 , 404) 4
Example 5. Find the HCF and LCM of 6, 72 and 120, using the prime factorisation
method.
Solution. We have, 6 = 2 × 3, 72 = 23 × 32, 120 = 23 × 3 × 5
1 1
Here, 2 and 3 are the smallest powers of the common factors 2 and 3
respectively.
So, HCF (6, 72, 120) = 21 × 31 = 2 × 3 = 6
23, 32 and 51 are the greatest powers of the prime factors 2, 3 and 5
respectively involved in the three numbers.
So, LCM (6, 72, 120) = 23 × 32 × 51 = 360
Remark: Notice, 6 × 72 × 120 ≠ HCF (6, 72, 120) × LCM (6, 72, 120).
So, the product of three numbers is not equal to the product of their
HCF and LCM.
Example 7. Check whether expansion of (252)n, n ∈ N can end with zero for any n as
natural number.
Solution. We know that the expansion of (x)n, n ∈ N can end with zero only if prime
factorisation of base number i.e., x has at least one 2 and one 5 otherwise
the expansion (x)n can never end with zero.
Now to check whether (252)n can end with zero or not we will find the
prime factors of base i.e., 252 which are as under
252 = 2 × 2 × 3 × 3 × 7 = 22 × 32 × 7
Real Numbers 7
Example 12. There is a circular path around a sports field. Sonia takes 18 minutes to
drive one round of the field, while Ravi takes 12 minutes for the same.
Suppose they both start at the same point and at the same time, and go
in the same direction. After how many minutes will they meet again at
the starting point? [NCERT Textbook]
Solution. They will be again at the starting point at least at common multiples of
18 and 12 minutes. To find the LCM of 18 and 12, we have
2 18 2 12
18 = 2 × 3 × 3 and 12 = 2 × 2 × 3
3 9 2 6
LCM of 18 and 12 = 2 × 2 × 3 × 3 = 36 3 3 3 3
1 1
So, Sonia and Ravi will meet again at the starting point after
36 minutes.
Example 13. A circular field has a circumference of 360 km. Two cyclists Aayush and
Anuj start together and can cycle at speeds of 12 km/hr and 15 km/hr,
respectively, round the circular field. After how many hours will they
meet again at starting point?
Solution. Speed of Aayush = 12 km/hr.
360
Number of hours taken by Aayush to complete one round = = 30
12
Speed of Anuj = 15 km/hr
360
Number of hours taken by Anuj to complete one round = = 24
15
So, Aayush and Anuj complete one round in 30 hrs and 24 hrs, respectively.
Now, let us find the LCM of 30 and 24.
30 = 2 × 3 × 5
24 = 23 × 3
Then, LCM (30, 24) = 23 × 3 × 5 = 120.
Hence, Aayush and Anuj will meet each other again after 120 hours.
Example 14. Four bells toll at an interval of 8, 12, 15 and 18 seconds respectively.
All the four bells begin to toll together. How many times will they toll
together in one hour excluding the one at the start?
Solution. LCM of 8,12,15 and 18 = 360
So, after each 360 seconds i.e., after each 6 minutes four bells will toll
together so in 1 hour i.e., in 60 minutes these four bells will toll together
60
= = 10 times
6
Real Numbers 9
Example 15. In a school there are two sections of class X and the sections are A and B.
There are 96 students in section A and 120 students in section B. Determine
the least number of books required for the library of the school so that
the books can be distributed equally among students of the section A or
that of the section B?
Solution. Least number of books required = LCM of 96 and 120
96 = 2 × 2 × 2 × 2 × 2 × 3 = 25 × 3
120 = 2 × 2 × 2 × 3 × 5 = 23 × 31 × 51
Hence, LCM (96, 120) = 25 × 31 × 51 = 480.
So, required number of books = 480.
Exercise 1.1
TYPE – 1: Questions based on Prime Factorisation
1. Find the prime factors of the following numbers:
(i) 176 (ii) 256 (iii) 4825 (iv) 12673
2. Find the missing numbers in the following factor trees.
(i) (ii)
3. Find the LCM and HCF of the following pairs of integers by applying the
Fundamental Theorem of Arithmetic method.
(i) 455, 78 (ii) 408, 170 (iii) 13, 11
4. Find the LCM and HCF of the following integers by applying the prime factorisation
method.
(i) 275, 225, 175 (ii) 765, 510, 408 (iii) 19, 13, 7
5. Given that HCF (1261, 1067) = 97, find LCM (1261, 1067). [CBSE 2020]
6. Find the LCM and HCF of the following pairs of integers and verify:
LCM × HCF = Product of the two numbers.
(i) 26 and 91 (ii) 510 and 92 (iii) 336 and 54
ANSWERS
1. (i) 176 = 2 × 2 × 2 × 2 × 11 (ii) 256 = 2 × 2 × 2 × 2 × 2 × 2 × 2 × 2
(iii) 4825 = 5 × 5 × 1931 (iv) 12673 = 19 × 23 × 29
2. (i) From top to bottom 15015, 5005, 7, 13
(ii) From top to bottom 3, 5, 1729, 247
3. (i) HCF = 13, LCM = 2730 (ii) HCF = 34, LCM = 2040
(iii) HCF = 1, LCM = 143
4. (i) HCF = 25, LCM = 17325, (ii) HCF = 51, LCM = 6120
(iii) HCF = 1, LCM = 1729
5. LCM = 13871 6. (i) LCM = 182, HCF = 13, (ii) LCM = 23460, HCF = 2,
(iii) LCM = 3024, HCF = 6
Real Numbers 11
7. No 8. No 9. No
10. Yes for n = 4 11. Yes for n = 4
12. Yes 13. Yes
14. (i) Prime (ii) Composite (iii) Prime (iv) Composite
15. 60 minutes 16. 36 minutes 17. (2520 cm)
18. 7 members 19. English = 2, Hindi = 5, Mathematics = 7
20. No. of rooms = 21.
Note: As we know that sum, difference, product and division of any two rational numbers
is always rational (for division rational number in denominator should not be zero).
Similarly, sum, difference, product and division of a rational number with an irrational
always results in an irrational number.
Theorem
Let p be a prime number. If p divides a2, then p divides a, where a is a positive
integer.
Proof. Let p1, p2, p3 ... pn be the factors of a.
⇒ a = p1. p2. p3 ... pn, where p1, p2, .... are primes not necessarily all distinct.
Therefore, a2 = (p1. p2. p3 ... pn) . (p1. p2. p3 .... pn)
⇒ a2 = p12. p22. p32 ... pn2
Give that, p divides a2 ... (1)
2
⇒ p is a prime factor of a , from the Fundamental Theorem of Arithmetic.
the prime factors of a2 are only p1, p2, p3 .... pn ... (2)
(Uniqueness of factors of Fundamental Theorem of Arithmetic)
From (1) and (2), we have
p is a prime factor of p1. p2. p3 .... pn
⇒ p divides a. Proved.
12 S. Chand’s New Mathematics for Class X
Solved Examples
But right now we have shown that ‘p’ and ‘q’ have 3 as common factor
so it contradicts our supposition that ÷3 is rational.
Hence, our supposition that ÷3 is rational is wrong.
Hence, we can conclude from this that ÷3 is an irrational number.
Let p=5m ⇒ p2 = 25 m2
Putting the value of p2 in (1), we get
25 m2 = 5 q2 ⇒ 5 m2 = q2
⇒ 5 divides q2 ⇒ 5 divides q. ... (3) (Concept 2)
Thus, from (2), 5 divides p and from (3), 5 also divides q. It means 5 is a
common factor of p and q. This contradicts the supposition so there is no
common factor of p and q.
Hence, ÷5 is an irrational number. Proved.
Example 19. Show that ÷7 is an irrational number. (Withous using contradiction
method)
Solution. Let us find out square root of 7 by division method.
2 . 6 4 5 7 5 1 3
2 7.00 00 00 00 00 00 00
2 –4
46 300
6 –276
524 2400
4 –2096
5285 30400
5 –26425
5 2 9 0 7 397500
7 –370349
5 2 9 1 4 5 2715100
5 –2645725
5 2 9 1 5 0 1 6937500
1 –5291501
5 2 9 1 5 0 2 3 1645999
(ii) Let us assume, to the contrary, that 7÷5 is rational. That is, we can
p
find co-prime integers p and q (q ≠ 0) such that 7 5 = .
q
p
So, 5= .
7q
p
Since, p and q are integers, is rational and so is ÷5.
7q
But this contradicts the fact that ÷5 is irrational.
So, we conclude that 7÷5 is an irrational. Proved.
3 p
Let 6 = , (where p and q are co-primes, q ≠ 0)
q
p3
=6 ... (1) (Taking cube on both sides)
q3
Now 13 = 1 and 23 = 8
p3 p
Thus, 1<6<8 ⇒ 1< 3
< 8 fi 1< <2
q q
3
p p3
From (1), =6 fi 6q 2 = ...(2)
q3 q
As q is an integer ⇒ 6q2 is an integer ...(3)
Since, p and q have no common factor.
⇒ p3 and q will have no common factor.
p3
⇒ is a fraction ...(4)
q
Real Numbers 17
Example 24. Prove that n is not a rational number, if n is not a perfect square.
Solution. n is a rational number.
Suppose
p
n=
q
(p and q have no common factor co-primes, and q ≠ 0)
p2
⇒ n= (Squaring both sides)
q2
⇒ p2 = nq2 ...(1)
2
⇒ n divides p
⇒ n divides p ... (2) (Concept 2)
Let p = nm
⇒ p2 = n2 m2 (Squaring both sides)
2
Putting the value of p in (1), we get
n2m2 = nq2
⇒ q2 = nm2
⇒ n divides q2
⇒ n divides q ... (3) (Concept 2)
From (2), n divides p and from (3) n divides q. It means n is a factor of
both p and q.
This contradicts the assumption that p and q have no common factor.
So our supposition is wrong. Hence, ÷n cannot be a rational number.
Proved.
Example 25. Prove that ÷2 + ÷3 is an irrational number. (NCERT Textbook)
Solution. Assume that ÷2 + ÷3 is a rational number ‘a’.
\ a= 2+ 3
⇒ a 2 = ( 2 + 3 )2 (Squaring both sides)
2
⇒ a = 2+3+2 2 3
⇒ a2 = 5 + 2 6
⇒ a2 - 5 = 2 6
a2 - 5
⇒ = 6 ...(1)
2
18 S. Chand’s New Mathematics for Class X
a2 - 5
Now is a rational number as ‘a’ is a rational number (by assumption)
2
From (1), ÷6 is a rational number. [\ LHS is rational so RHS will be rational]
But ÷6 is an irrational number. (Example 24)
Thus, there is a contradiction.
Hence, our assumption is wrong.
Therefore, ÷2 + ÷3 is an irrational number. Proved.
Exercise 1.2
1. Prove that ÷5 is an irrational number.
2. Prove that 2 + ÷6 is an irrational number.
3. Prove that ÷7 – 8 is an irrational number.
4. Prove that 3÷11 is an irrational number.
5
5. Prove that is an irrational number.
2
6. Prove that (÷3 – ÷8) is an irrational number.
7. Prove that ÷13 + ÷17 is an irrational number.
8. Prove that 3÷10 is an irrational number.
9. Give rational approximation of ÷5 correct to two decimal places.
10. Identify the following as rational or irrational.
9
(i) ÷4 (ii) 3÷18 (iii) ÷1.44 (iv)
27
(v) –÷0.64 (vi) ÷100 (vii) (÷3 + 1)2 (viii) (÷5 + 1)(÷5 – 1)
6
(ix) ( 3 + 4 )( 5 + 6 ) (x)
2 3
ANSWERS
9. 2.23
10. (i) Rational Number (ii) Irrational Number
(iii) Rational Number (iv) Irrational Number
(v) Rational Number (vi) Rational Number
(vii) Irrational Number (viii) Rational Number
(ix) Irrational Number (x) Irrational Number.
Real Numbers 19
REVISION EXERCISE
section – a
section – B
Very Short Answer Type Questions (2 Marks each)
1. Find the LCM of (306, 1314), m
section – C
Short Answer Type Questions (3 Marks each)
1. Prove that 3 + ÷7 is an irrational number.
2. Prove that 2÷3 – 4 is an irrational number.
3. A merchant has 120 litres of oil of one kind, 180 litres of another kind and
240 litres of third kind. He wants to sell the oil by filling the three kinds of oil in
tins of equal capacity. What should be the greatest capacity of such a tin?
Ans. 60 litres
4. Find the HCF and LCM of 306 and 54. Verify that HCF × LCM = Product of the
two numbers. Ans. HCF = 18, LCM = 918
5. Find the LCM and HCF of 15, 18, 45 by the prime factorisation method.
Ans. LCM = 90, HCF = 3
1
6. Prove that is an irrational number.
2+ 3
7. Show that 9n cannot end with 2 for any integer n.
8. Prove that 3 + 5 is an irrational number.
9. Prove that 2 3 - 7 is an irrational number.
10. Prove that (5 - 3 ) is an irrational number.
⇒ LCM = 22338
n
5. Check whether 6 can end with the digit 0 for any natural number n.
Sol. Page 7 Example 10
N
6. Explain why 7 × 11 × 13 + 13 and 7 × 6 × 5 × 4 × 3 × 2 × 1 + 5 are composite numbers.
C
Sol. Page 7 Example 11
E
7. There is a circular path around a sports field. Sonia takes 18 minutes to drive one
R
round of the field, while Ravi takes 12 minutes for the same. Suppose they both T
start at the same point and at the same time, and go in the same direction. After
how many minutes will they meet again at the starting point? e
Sol. Page 8 Example 12 x
NCERT Exercise 1.2
e
m
1. Prove that ÷5 is irrational. p
Sol. Page 13 Example 18 l
2. Prove that 3 + 2÷5 is irrational. a
Sol. Page 15 Example 21 r
3. Prove that following are irrationals
1
(i) (ii) 7÷5 (iii) 6 + ÷2
2
Sol. Page 15 Example 22
SOLUTIONs FOR NCERT exemplar EXERCISES
NCERT exemplar Exercise 1.1
5. The largest number which divides 70 and 125, leaving remainder 5 and 8
respectively is:
(a) 13 (b) 65 (c) 875 (d) 1750
6. (b) If two positive integers ‘a’ and ‘b’ are written as a = x3y2 and b = xy3, where x, y are
prime numbers. HCF of ‘a’ and ‘b’ is:
(a) xy (b) xy2 (c) x3y3 (d) x2y2
3 2
Sol. Prime factors of a = x y = x × x × x × y × y
Prime factors of b = xy3 = x × y × y × y
So, HCF = Product of common factors = xy2
Hence, option (b) is correct.
7. For two positive integers p = ab2 and q = a3b LCM is [a and b are prime numbers]
(a) ab (b) a2b2 (c) a3b2 (d) a3b3
Sol. (c) Clearly p = a × b × b and q = a × a × a × b
So, LCM will be a3 × b2
Hence, option (c) is correct.
8. The product of a non-zero rational nuimber and an irrational number is:
(a) always irrational (b) always rational
(c) rational or irrational (d) one
Sol. (a) It is true.
24 S. Chand’s New Mathematics for Class X
9. The least number that is divisible by all the numbers from 1 to 10 (both inclusive)
is:
N (a) 10 (b) 100 (c) 504 (d) 2520
C Sol. (d) Clearly such least number is LCM of 1, 2, 3, 4, 5, 6, 7, 8, 9 and 10.
E Which is 2520.
R Hence, option (d) is correct.
T
NCERT exemplar Exercise 1.2
e 6. The numbers 525 and 3000 are both divisible only by 3, 5, 15, 25 and 75. What is
x HCF of (525, 3000)? Justify your answer.
e Sol. Since, the HCF (525, 3000) = 75
m and the numbers 3, 5, 15, 25 and 75 divides the numbers 525 and 3000 that mean
these terms are common in both 525 and 3000. So, the highest common factor
p among these is 75.
l 7. Explain why 3 × 5 × 7 + 7 is composite number.
a Sol. Value of 3 × 5 × 7 + 7 = 112
r 112 can be written as (112) = 24 × 7
As 112 can be expressed as product of two or more prime number hence,
3 × 5 × 7 + 7 is composite number.
8. Can two numbers have 18 as their HCF and 380 as their LCM? Give reason.
Sol. No, because 380 is not divisible by 18.
Rule: LCM is always divisible by HCF.
NCERT exemplar Exercise 1.3
10. Prove that ÷3 + ÷5 is irrational.
Sol. If possible let ÷3 + ÷5 is rational. So we can say ÷3 + ÷5 = r {where ‘r’ is rational
number}
As ÷3 + ÷5 = r ... (1)
On squaring both sides of equation (1)
We get 8 + 2÷15 = r2
r2 - 8
⇒ ÷15 = ... (2)
2
Clearly RHS of equation (2) is rational and LHS is irrational.
So, our supposition is wrong.
Hence, ÷3 + ÷5 is irrational.
11. Show that (12)n cannot end with digit ‘0’ or ‘5’ for any natural number ‘n’.
(CBSE 2020)
Sol. For a number to end with digit zero its prime factors must have atleast one 2 and
one 5 both and for a number to end with 5 its prime factors must have at least
one 5.
Real Numbers 25
Therefore, q =a– p
On squaring both sides, we get
q = a2 + p – 2a p [Q (a – b)2 = a2 + b2 – 2ab]
a2 + p - q
Therefore, p = , which is a contraction as the right-hand side is a
2a
rational number while p is irrational, since p and q are prime numbers.
Hence, p + q is irrational.
CASE STUDIES
Case Study 1
Resident welfare association
of a town decided to create
a library for the children of
their locality to enhance the
How do we know what
interest of children toward number to look for?
studies and learning moral
values. They put 12 books of
Vedic Mathematics, 18 books
of Social Studies, 30 books of
General Science and 42 Books
of Moral Education.
26 S. Chand’s New Mathematics for Class X
In the library at one time if equal number of children comes to read the books of Vedic
Mathematics, Social Studies, General Science and Moral Education at same time as
decided by R.W.A. All these books are of same size and will be kept in minimum
number of shelves in the library so that each shelf contains equal number of books and
of same subject.
1. What is the maximum number of students of each subject comes to study in library?
(a) 2 (b) 4 (c) 6 (d) 8.
2. Number of shelves required to keep books of Moral Education
(a) 2 (b) 3 (c) 5 (d) 7
3. Total number of shelves-required to keep all the books
(a) 17 (b) 15 (c) 12 (d) 10.
4. If cost of each shelf is ` 500, then total cost of all shelf required is Rs
(a) ` 8000 (b) ` 8500 (c) ` 9000 (d) ` 10000.
Solution:
1. Maximum number of students of each subject comes to study in library = HCF
of 12, 18, 30 and 42, that is 6.
2. In one shelf we can keep 6 books, so number of shelves required to keep books
of Moral Education = 42/6 = 7.
3. Total number of books are 12 + 18 + 30 + 42 = 102, hence total number of shelves
required to keep all the books = 102/6 = 17.
4. Total cost of all shelves =17 x 500 = ` 8500.
ANSWERS
1. (c) 2. (d) 3. (a) 4. (b)
Case Study 2
There are 110 students in a school out of these 20 students scored marks more than or
equal to 95% levelled section-A, 25 students scored marks more than or equal to 90% but
less than 95% levelled section-B, 30 students scored marks more than or equal to 85%
but less than 90% levelled section-C and 35 students scored marks more than or equal
to 80% but less than 85% levelled
section-D. School decided to award
all these students with suitable
prizes as per their percentage they
scored. To save the time of award
ceremony students are divided in
the groups of maximum and equal
number of students in each group
and no student of any section can
be grouped with other section.
ANSWERS
1. (b) 2. (d) 3. (b) 4. (c)
Case Study 3
We know that the sum, difference, product and division of
two rational numbers are always a rational number and sum,
difference, product and division of one rational number and
other irrational number is always an irrational number.
Answer the following questions:
1. Product of 2 and ÷3 is
(a) rational number (b) irrational number
(c) ÷12 (d) both b and c.
2. Difference of –8 and ÷8 is
(a) rational number (b) irrational number
(c) –8 –2 ÷2 (d) both b and c.
3. Division of ÷12 and ÷3 is
(a) rational number (b) irrational number
(c) 2 (d) both a and c.
4. Sum of 3 + ÷2 and 3 –÷2 is
(a) rational number (b) 6
(c) ÷4 (d) both a and b.
Solution:
1. As 2 is a rational number and ÷3 is an irrational number so their product is an
irrational number which is equal to ÷12 also.
2. As –8 is a rational number and ÷8 is an irrational number so their difference will
be an irrational number which is equal to –8 –2÷2 also.
3. As division of ÷12 and ÷3 is ÷4 = 2, which is a rational number.
4. As sum of 3 +÷2 and 3 –÷2 is 6, which is a rational number.
28 S. Chand’s New Mathematics for Class X
ANSWERS
1. (d) 2. (d) 3. (d) 4. (d)
Case Study 4
A Mathematics Exhibition is being conducted in your School and one of your friends
is making a model of a factor tree. She has some difficulty and asks for your help in
completing a quiz for the audience.
Observe the following factor tree and answer the following: (CBSE Question Bank)
x
5 2783
y 253
11 z
ANSWERS
1. (b) 2. (c) 3. (b) 4. (a) 5. (c)
O A B
7. They all will meet again at Points A and B at 12:24 pm. As LCM of 3, 6, 9, 8, 12
and 16 is 144 minutes which is equal to 2 hours and 24 minutes.
ANSWERS
1. 10 : 18 a.m 2. 6 3. 10 : 48 a.m 4. 6 5. 3
6. 6 7. 12 : 24 p.m
WORKSHEET # 1
FOR PRACTISING THE PROBLEMS BASED ON TRUE/FALSE
Say True (T) or False (F):
1. LCM of two or more numbers is always divisible by their HCF.
2. 7 × 11 × 13 + 13 is a prime number.
3. HCF of two prime numbers is always 1.
n
4. (38) can end with zero.
5. Every real number is always rational.
6. Product of a non-zero rational and an irrational is always an irrational number.
7. Reciprocal of an irrational number is always an irrational number.
8. There are infinitely many rational numbers between any two irrational numbers.
9. Product of two prime numbers is always equal to their LCM.
10. There is no irrational number between any two rational numbers.
11. HCF of two numbers can be 18 if their LCM is 380.
12. LCM of two co-prime numbers is always 1.
ANSWERS
1. T 2. F 3. T 4. F
5. F 6. T 7. T 8. T
9. T 10. F 11. F 12. T
WORKSHEET # 2
FOR PRACTISING THE PROBLEMS BASED ON
FILL IN THE BLANKS
ANSWERS
1. 2 2. fractional number 3. 5
4. unique 5. irrational 6. a × b
7. rational and irrational 8. 1 9. real number
1
10. 1 and
6
WORKSHEET # 3
FOR PRACTISING THE PROBLEMS BASED ON
MATCH THE COLUMN
ANSWERS
1. (b) 2. (d) 3. (c) 4. (e) 5. (a)
32 S. Chand’s New Mathematics for Class X
WORKSHEET # 4
FOR PRACTISING THE PROBLEMS BASED ON
ASSERTION REASONING
Directions: The following questions consist of two statements, one labelled the
Assertion (A) and the other labelled as Reason (R). You are to examine these two
statements carefully and decide if the Assertion (A) and the Reason (R) are individually
true and if so, whether the Reason is a correct explanation of the item, Assertion. Select
your answers of these questions using the codes given below and mark your answer
sheet accordingly.
Codes:
(a) Both (A) and (R) are true and (R) is the correct explanation of (A).
(b) Both (A) and (R) are true and (R) is the incorrect explanation of (A).
(c) (A) is true but (R) is false.
(d) (A) is false but (R) is true.
1. Assertion : LCM and HCF of two numbers are 105 and 5. If one of the number
is 15 then other will be 35.
Reason : LCM × HCF = Product of two numbers.
2. Assertion : LCM of two numbers is always a factor of their HCF.
Reason : LCM × HCF = Product of two number
3. Assertion : Factors of every composite number are unique.
Reason : Fundamental Theorem of Arithmetic
4. Assertion : (÷2 + ÷5) is irrational
Reason :
Sum and difference of two irrational numbers can be rational
number or can be irrational number.
5. Assertion : p is an irrational number.
Reason :
Ratio of circumference and diameter is always approximate value,
never exact.
ANSWERS
1. (a) 2. (d) 3. (a) 4. (b) 5. (a)
WORKSHEET # 5
FOR PRACTISING THE PROBLEMS BASED on
MCQs WITH MORE THAN ONE CORRECT OPTIONS
1. There is no number between 0.9 and 1
(a) Rational (b) Irrational (c) Real number (d) None of these
n
2. Numbers in the form of (4) , n ∈ N can end with the digit
(a) 5 (b) 4 (c) 1 (d) 6
Real Numbers 33
a
3. If a = ÷12 and b = ÷3 then is a/an
(a) rational number
b (b) irrational number
(c) integer (d) natural number
4. Number ÷3 + ÷2 is not a/an
(a) whole number (b) rational number
(c) integer (d) irrational number
5. Number 2.375 is
(a) rational number (b) irrational number
(c) non-terminating repeating decimal (d) None of these
6. If HCF of two numbers is 1 then two numbers can be
(a) prime (b) coprime (c) cannot say (d) none of these
7. LCM of numbers 1, 2, 3 is equal to their
(a) product (b) division (c) sum (d) None
ANSWERS
1. (a), (b) and (c) 2. (b) and (d) 3. (a), (c) and (d)
4. (a), (b) and (c) 5. (a) and (c) 6. (a) and (b)
7. (a) and (c)
a
10. A number which cannot be expressed in the form of , a, b Œ I, b ≠ 0 is known
b
as an irrational number.
1
11. If a be an irrational number, then – a and are also irrational.
a
12. If ab be an irrational number then a + b is also irrational.
14. If a be an irrational number and r a non-zero rational number, then the sum,
difference, product and quotient of r and a are also irrational.
L.C.M. of Numerators
15. L.C.M. of fractions or Rational numbers =
H.C.F. of Denominators
H.C.F. of Numerators
16. H.C.F. of fractions or Rational numbers =
L.C.M.of Denominators